Solved papers for JAMIA MILLIA ISLAMIA Jamia Millia Islamia Solved Paper-2010

done Jamia Millia Islamia Solved Paper-2010 Total Questions - 170

  • question_answer1) A car leaves station\[X\]for station Y every 10 min. The distance between\[X\]and Y is 60 km. The car travels at a speed of 60 km/h. A man drives a car from Y station towards\[X\]station at speed 60 km/h. If he starts at the moment when one of the car leaves station\[X\]how many cars would be meet on route?

    A)
     10                                         

    B)
     11

    C)
     20                                         

    D)
     21

    View Answer play_arrow
  • question_answer2)     A ball rolls off the stairway with a horizontal velocity of magnitude 1.8 m/s. The steps are 0.20 m high and 0.20 m wide. Which step will the ball hit first?

    A)
     First                                     

    B)
     Second

    C)
     Third                                    

    D)
     Fourth

    View Answer play_arrow
  • question_answer3)     A sphere of mass 0.20 kg is attached to an inextensible string of length 0.5m whose upper end is fixed to the ceiling. The sphere is made to describe a horizontal circle of radius 0.3 m. The speed of the sphere will be

    A)
     1.5 m/s                               

    B)
     2.5 m/s

    C)
     3.2 m/s               

    D)
     4.7 m/s

    View Answer play_arrow
  • question_answer4)     Steady rain giving 5 mm an hour, turns suddenly into a downpour 20 mm an hour and the speed of rain drops falling vertically on to a flat roof simultaneously doubles. The pressure exerted by the falling rain on the roof rises by a factor of

    A)
     2                                            

    B)
    \[2\sqrt{2}\]

    C)
     \[4\sqrt{2}\]                                     

    D)
     8

    View Answer play_arrow
  • question_answer5)     An open knife edge of mass M is dropped from a height h on a wooden floor. If the blade penetrates distance into the wood, the average resistance offered by the wood to the blade is

    A)
     \[Mg\]                                

    B)
     \[Mg\left( 1+\frac{h}{s} \right)\]

    C)
     \[Mg\left( \frac{1-h}{s} \right)\]                             

    D)
     \[Mg{{\left( \frac{1+h}{s} \right)}^{2}}\]

    View Answer play_arrow
  • question_answer6)     A particle moves in\[X-Y\]plane under the influence of a force\[\overrightarrow{F}\]such that its instantaneous momentum is\[\overrightarrow{P}=\hat{i}2\cos t+\hat{j}2\sin t\]. What is the angle between the force and instantaneous momentum?

    A)
     \[0{}^\circ \]                                     

    B)
     \[180{}^\circ \]

    C)
     \[90{}^\circ \]                                  

    D)
     \[45{}^\circ \]

    View Answer play_arrow
  • question_answer7)     The KE of a body varies directly as the time (t) elapsed. The force acting varies directly as

    A)
     \[{{t}^{-1}}\]                                    

    B)
     \[{{t}^{-1/2}}\]

    C)
     \[{{t}^{1/2}}\]                                  

    D)
     \[t\]

    View Answer play_arrow
  • question_answer8)     A gas at the temperature 250 K is contained in a closed vessel. If the gas is heated through 1 K, then the percentage increase in its pressure will be

    A)
     0.4%                                    

    B)
     0.2%

    C)
     0.1%                                    

    D)
     0.8%

    View Answer play_arrow
  • question_answer9)     A flywheel rotates with a uniform angular acceleration. Its angular velocity increases from 20 n rad/s to 40 n rad/s in 10 s. How many rotations did it make in this period?

    A)
     80                                         

    B)
     100

    C)
     120                                       

    D)
     150

    View Answer play_arrow
  • question_answer10)     A point P lies on the axis of a ring of mass M and radius R at a distance 2R from its centre\[O\] A small particle starts from P and reaches Q under gravitational attraction only. Its speed at \[O\]will be

    A)
     zero                                     

    B)
     \[\sqrt{\frac{2GM}{R}}\]

    C)
     \[\sqrt{\frac{2GM}{R}(\sqrt{5}-1)}\]     

    D)
     \[\sqrt{\frac{2GM}{R}\left( 1-\frac{1}{\sqrt{5}} \right)}\]

    View Answer play_arrow
  • question_answer11)     Two vertical glass plates 1 mm apart are dipped into water. How high will be the water rise between the plates, if the surface tension of water is\[70\text{ }dyne/c{{m}^{2}}\]?

    A)
     2.86cm                                

    B)
     1.43cm

    C)
     5.72cm                                

    D)
     1.63cm

    View Answer play_arrow
  • question_answer12)     A steel ball is dropped from a height h and it makes a perfectly elastic collision with a horizontal plane. Following its initial release, it will make periodic motion with frequency

    A)
     \[2\pi \sqrt{\frac{g}{h}}\]                           

    B)
     \[2\pi \sqrt{\frac{h}{g}}\]

    C)
     \[\frac{1}{2}\sqrt{\frac{g}{2h}}\]                             

    D)
     \[\frac{1}{2}\sqrt{\frac{2h}{g}}\]

    View Answer play_arrow
  • question_answer13)     Two closed organ pipes A and B have the same length, A is wider than B. They resonate in the fundamental mode at frequencies\[{{n}_{A}}\]and\[{{n}_{B}}\] respectively

    A)
     \[{{n}_{A}}={{n}_{B}}\]

    B)
     \[{{n}_{A}}>{{n}_{B}}\]

    C)
     \[{{n}_{A}}<{{n}_{B}}\]

    D)
     Either  or  depending on the ratio of their diameters

    View Answer play_arrow
  • question_answer14)     Shown below is a distribution of charges. The flux of electric field due to these charges through the surface is

    A)
     \[\frac{3q}{{{\varepsilon }_{0}}}\]                                          

    B)
     zero

    C)
     \[\frac{2q}{{{\varepsilon }_{0}}}\]                                          

    D)
     \[\frac{q}{{{\varepsilon }_{0}}}\]

    View Answer play_arrow
  • question_answer15)     A parallel plate capacitor is connected across a 2 V battery and charged. The battery is then disconnected and a glass slab is introduced between plates. Which of the following pairs of quantities decrease?

    A)
     Charge and potential difference

    B)
     Potential difference and energy stored

    C)
     Energy stored and capacitance

    D)
     Capacitance and charge

    View Answer play_arrow
  • question_answer16)     Four resistances of\[10\,\Omega ,60\,\Omega ,100\,\Omega \]and\[200\,\Omega \]. respectively taken in order are used to form a Wheatstones bridge. A 15 V battery is connected to the ends of a\[200\,\Omega \] resistance. The current through it will be

    A)
     \[7.5\times {{10}^{-5}}A\]          

    B)
     \[7.5\times {{10}^{-4}}A\]

    C)
     \[7.5\times {{10}^{-3}}A\]          

    D)
     \[7.5\times {{10}^{-2}}A\]

    View Answer play_arrow
  • question_answer17)     Magnetic field at the centre of a circular loop of area A is B. The magnetic moment of the loop will be

    A)
     \[\frac{B{{A}^{2}}}{{{\mu }_{0}}\pi }\]                  

    B)
     \[\frac{B{{A}^{3/2}}}{{{\mu }_{0}}\pi }\]

    C)
     \[\frac{B{{A}^{3/2}}}{{{\mu }_{0}}{{\pi }^{1/2}}}\]                           

    D)
     \[\frac{2B{{A}^{3/2}}}{{{\mu }_{0}}{{\pi }^{1/2}}}\]

    View Answer play_arrow
  • question_answer18)     A thin magnet is cut into two equal parts by cutting it parallel to its length. If the original time period of vibration is 4 s the time period of each part in the same field will be

    A)
     \[4s\]                                  

    B)
     \[2s\]

    C)
     \[4\sqrt{2}s\]                                   

    D)
     None of these

    View Answer play_arrow
  • question_answer19)     The moment of magnet is\[0.1\text{ }A{{m}^{2}}\]and the force acting on each pole in a uniform magnetic field of 0.36 oersted is\[1.44\times {{10}^{-4}}N\]. The distance between the poles of magnet is

    A)
     2.5 cm                                 

    B)
     5.0 cm

    C)
     1.25cm                                

    D)
     1.17cm

    View Answer play_arrow
  • question_answer20)     A solenoid 600 mm long has 50 turns on it and is wound on an iron rod of 7.5 mm radius. Find the flux through the solenoid when the current in it is 3 A. The relative permeabillity of iron is 600.

    A)
     1.66 Wb                              

    B)
     1.66 N Wb

    C)
     1.66 m Wb     

    D)
    \[1.66\text{ }\mu \,Wb\]

    View Answer play_arrow
  • question_answer21)     A transformer has an efficiency of 80%. It works at 4 kW and 100 V. If secondary voltage is 240 V the current in primary coil is

    A)
     0.4 A                                    

    B)
     4 A

    C)
     10 A                                     

    D)
     40 A

    View Answer play_arrow
  • question_answer22)     sIn a plane electromagnetic wave, the electric field oscillates sinusoid ally at a frequency \[2\times {{10}^{10}}Hz\]and amplitude 48V/m. The wavelength of the wave is      

    A)
     \[24\times {{10}^{-10}}m\]        

    B)
     \[1.5\times {{10}^{-2}}m\]

    C)
     \[4.16\times {{10}^{8}}m\]        

    D)
     \[3\times {{10}^{8}}m\]

    View Answer play_arrow
  • question_answer23)     A parallel beam of monochromatic unpolarised light is incident on a transparent dielectric plate of refractive indie\[\frac{1}{\sqrt{3}}\]. The reflected beam is completely polarised. Then the angle of incidence is

    A)
     \[30{}^\circ \]                                  

    B)
     \[60{}^\circ \]

    C)
     \[45{}^\circ \]                                  

    D)
     \[75{}^\circ \]

    View Answer play_arrow
  • question_answer24)     A microscope is focused on a mark on a piece of paper and then a slab of glass of thickness 3 cm and refractive index 1.5 is placed over the mark. How should the microscope be moved to get the mark again in focus?

    A)
     2 cm upward   

    B)
     1 cm upward

    C)
     4.5 cm downward,         

    D)
     1 cm downward

    View Answer play_arrow
  • question_answer25)     The wavelength of 1 keV photon is\[1.24\times {{10}^{-9}}\] m What is the frequency of 1 MeV photon?

    A)
     \[2.4\times {{10}^{15}}Hz\]       

    B)
     \[2.4\times {{10}^{20}}Hz\]

    C)
     \[1.24\times {{10}^{15}}Hz\]     

    D)
     \[1.24\times {{10}^{20}}Hz\]

    View Answer play_arrow
  • question_answer26)     An electron revolving in an orbit of radius 0.5 A in a hydrogen atom executes\[{{10}^{16}}rev/s\]. The magnetic moment of electron due to its orbital motion will be,

    A)
    \[1256\times {{10}^{-26}}A{{m}^{2}}\]

    B)
     \[6.53\times {{10}^{-26}}A{{m}^{2}}\]

    C)
     zero

    D)
     \[256\times {{10}^{-26}}A{{m}^{2}}\]

    View Answer play_arrow
  • question_answer27)     The ratio of electron and hole currents in a semiconductor is 7/4 and the ratio of drift velocities of electrons and holes is 5/4 then ratio of concentrations of electrons and holes will be

    A)
     5/7                                       

    B)
     7/5

    C)
     25/49                                   

    D)
     49/25

    View Answer play_arrow
  • question_answer28)     In a common base transistor circuit, the current gain is 0.98 on changing emitter current by 5 mA, the change in collector current is

    A)
     0.196mA            

    B)
     2.45mA

    C)
     4.9mA   

    D)
     5.1mA

    View Answer play_arrow
  • question_answer29)     A bus starts moving with acceleration\[2\text{ }m/{{s}^{2}}\]. A cyclist 96 m behind the bus starts simultaneously towards the bus of 20 m/s. After what time will he be able to overtake the bus?

    A)
     \[4s\]                                  

    B)
     \[8s\]

    C)
     \[12s\]                                

    D)
     \[16s\]

    View Answer play_arrow
  • question_answer30)     Geostationary satellite orbits around the earth in a circular orbit of radius 36000 km. Then the time period of a spy satellite orbiting a 500 km above the earths surface (radius of earth = 6400 km) will approximately be

    A)
     \[\frac{1}{2}h\]                               

    B)
     \[1\,h\]

    C)
     \[2\,h\]                                              

    D)
     \[4\,h\]

    View Answer play_arrow
  • question_answer31)     The upper end of a wire 1 m long and 2 mm radius is clamped. The lower end is twisted through an angle of\[45{}^\circ \]. The angle of shear is

    A)
     \[0.09{}^\circ \]                               

    B)
     \[0.9{}^\circ \]

    C)
     \[9{}^\circ \]                                     

    D)
     \[90{}^\circ \]

    View Answer play_arrow
  • question_answer32)     An air filled parallel plate capacitor charged to potential\[{{V}_{1}}\]is connected to an uncharged identical parallel plate capacitor with dielectric constant K. The common potential is\[{{V}_{2}}\]. The value of K is

    A)
     \[\frac{{{V}_{1}}-{{V}_{2}}}{{{V}_{1}}+{{V}_{2}}}\]                          

    B)
     \[\frac{{{V}_{1}}}{{{V}_{1}}-{{V}_{2}}}\]

    C)
     \[\frac{{{V}_{1}}-{{V}_{2}}}{{{V}_{2}}}\]                               

    D)
     \[\frac{{{V}_{1}}-{{V}_{2}}}{{{V}_{1}}}\]

    View Answer play_arrow
  • question_answer33)     Three charged particles are initially in position\[-1\]. They are free to move and they come to position-2, after some time. Let\[{{U}_{1}}\]and\[{{U}_{2}}\]be the electrostatic potential energies in position\[-1\]and 2 then

    A)
     \[{{U}_{1}}={{U}_{2}}\]               

    B)
     \[{{U}_{2}}\ge {{U}_{1}}\]

    C)
     \[{{U}_{2}}>{{U}_{1}}\]               

    D)
     \[{{U}_{1}}>{{U}_{2}}\]

    View Answer play_arrow
  • question_answer34)     In Youngs double slit experiment one slit is covered with red filter and another slit is covered by green filter, then interference pattern will be

    A)
     red                                       

    B)
     green

    C)
     yellow                 

    D)
     invisible

    View Answer play_arrow
  • question_answer35)     Two plano-convex lenses of radius of curvature R and refractive index\[\mu =1.5\]will have equivalent focal length equal to R when they are placed

    A)
     at distance R/4

    B)
     at distance R/2

    C)
     at distanced

    D)
     in contact with each other

    View Answer play_arrow
  • question_answer36)     If energy E, velocity v and time T were taken as fundamental units, the dimensions for surface tension in these units are

    A)
     \[[{{E}^{-2}}{{V}^{1}}{{T}^{-2}}]\]           

    B)
     \[[{{E}^{-2}}{{V}^{-2}}{{T}^{1}}]\]

    C)
     \[[{{E}^{1}}{{V}^{-2}}{{T}^{-2}}]\]           

    D)
     \[[{{E}^{-2}}{{V}^{-2}}{{T}^{-2}}]\]

    View Answer play_arrow
  • question_answer37)     The angle between\[(\overrightarrow{A}\times \overrightarrow{B})\]and\[(\overrightarrow{A}+\overrightarrow{B})\]is

    A)
     \[\frac{\pi }{4}\]                                             

    B)
     \[\frac{\pi }{2}\]

    C)
     \[\pi \]                                                

    D)
     zero

    View Answer play_arrow
  • question_answer38)     Two stones are projected with the same velocity but making different angles with the horizontal. Their ranges are equal. If the angle of projection of one is\[\frac{\pi }{3}\]and maximum height is\[{{y}_{1}}\], then the maximum height of other will be

    A)
     \[3{{y}_{1}}\]                                   

    B)
     \[2{{y}_{1}}\]

    C)
     \[\frac{{{y}_{1}}}{2}\]                                   

    D)
     \[\frac{{{y}_{1}}}{3}\]

    View Answer play_arrow
  • question_answer39)     A disc of mass 10 g is kept horizontally in air by firing bullets of mass 5 g each at the rate of 10 per second. If the bullets rebound with the same speed, what is the velocity with which the bullets are fired?

    A)
     49cm/s                               

    B)
     98 cm/s

    C)
     147 cm/s            

    D)
     196 cm/s

    View Answer play_arrow
  • question_answer40)     A sphere of mass 100 g is attached to an inextensible string of length 1.3m whose upper end is fixed to the ceiling. The sphere is made to describe a horizontal circle of radius 50 cm. The time period of the revolution is

    A)
     5.0s                                      

    B)
     4.5s

    C)
     3.2s                                      

    D)
     2.3s

    View Answer play_arrow
  • question_answer41)     A uniform rod of length\[l\]is free to rotate in a vertical plane about a fixed horizontal axis through point B. The rod begins rotating from rest from its unstable equilibrium position. When it has turned through an angle\[\theta \]its angular velocity co is given as

    A)
     \[\sqrt{\frac{6g}{l}}\sin \theta \]                             

    B)
     \[\sqrt{\frac{6g}{l}}\sin \frac{\theta }{2}\]

    C)
     \[\sqrt{\frac{6g}{l}}\cos \frac{\theta }{2}\]                         

    D)
     \[\sqrt{\frac{6g}{l}}\cos \theta \]

    View Answer play_arrow
  • question_answer42)     The displacement of a particle executing periodic motion is given by \[y=4{{\cos }^{2}}\frac{t}{2}\sin 1000t\] The expression may be considered to be a result of superposition of ......... independent harmonic motions.

    A)
     1                                            

    B)
     3

    C)
     4                                            

    D)
     5

    View Answer play_arrow
  • question_answer43)     Oxygen gas is contained in a cylinder of volume \[1\times {{10}^{-3}}{{m}^{3}}\]at a temperature of 320 K and a pressure of\[3.53\times {{10}^{5}}N/{{m}^{2}}\]. After some of the oxygen is used at constant temperature, the pressure falls to\[2.7\times {{10}^{5}}N/{{m}^{2}}\]. The mass of oxygen used is

    A)
     0.01 kg                                

    B)
     0.001 kg

    C)
     0.002 kg                              

    D)
     0.003 kg

    View Answer play_arrow
  • question_answer44)     A reversible engine working between the temperature limits of 600 K and 1200K receives 50 kJ of heat. The work done by the engine will be

    A)
     50 kJ                                    

    B)
     100 kJ

    C)
     25 kJ                                    

    D)
     \[-25\text{ }kJ\]

    View Answer play_arrow
  • question_answer45)     For the stationary wave\[y=4\sin \frac{\pi x}{15}\cos 96\pi t\] The distance between a node and the next antinode is

    A)
     7.5cm                                  

    B)
     15cm

    C)
     22.5cm                                

    D)
     30cm

    View Answer play_arrow
  • question_answer46)     Two waves each of frequency 540 Hz travel at a speed of 330 m/s. If the source are in phase, in the beginning, the phase difference of the waves at a point 4 m from one source and 4.4 m from the other is

    A)
     \[59{}^\circ \]                                  

    B)
     \[118{}^\circ \]

    C)
     \[177{}^\circ \]                                

    D)
     \[236{}^\circ \]

    View Answer play_arrow
  • question_answer47)     Two point charges\[+36\mu C\]and\[-16\mu C\]are placed 10 cm apart. At a point\[X\], there is no resultant force on unit positive charge. The distance of\[X\]from\[+36\mu C\]charge is

    A)
     10cm                                   

    B)
     20cm

    C)
     30 cm                                  

    D)
     35 cm

    View Answer play_arrow
  • question_answer48)     Five resistors are connected between points\[X\]and V as shown in the diagram. A current of 10 A flows in the network from\[X\]to\[Y\].

    A)
     The potential difference between X and Z is equal to that between Z and Y

    B)
     The potential difference between X and Z is more than that between Z and Y

    C)
     The potential difference between X and Z is less than that between Z and Y

    D)
     The potential difference between Z and Y is 24V

    View Answer play_arrow
  • question_answer49)     The resistance of a galvanometer is\[25\,\Omega \], and it requires\[50\,\mu A\]for full deflection. The value of the shunt resistance required to convert it into an ammeter of 5 A is

    A)
     \[2.5\times {{10}^{-4}}\Omega \]           

    B)
     \[1.25\times {{10}^{-3}}\Omega \]

    C)
     \[0.05\,\Omega \]                         

    D)
     \[2.5\,\Omega \]

    View Answer play_arrow
  • question_answer50)     In circuit shown in figure if both the bulbs\[{{B}_{1}}\]and\[{{B}_{2}}\]are identical

    A)
     their brightness will be same

    B)
     \[{{B}_{1}}\]will be brighter than\[{{B}_{2}}\]

    C)
     as frequency of supply voltage is increased, brightness of\[{{B}_{1}}\]will increase and that of\[{{B}_{2}}\]will decrease

    D)
     only\[{{B}_{2}}\]will glow because the capacitor has infinite impedance

    View Answer play_arrow
  • question_answer51)     A certain radioactive substance has a half-life of 5 yr. Thus, for a nucleus in a sample of the element, the probability of decay in 10 yr is

    A)
     50%                                      

    B)
     75%

    C)
     100%                                   

    D)
     60%

    View Answer play_arrow
  • question_answer52)     An\[L-C\]resonant circuit contains a 400 pF capacitor and a\[100\,\mu H\]inductor. It is set into oscillation coupled to an antenna. The wavelength of the radiated electromagnetic waves is

    A)
     377 mm                              

    B)
     377 m

    C)
     377cm                                 

    D)
     3.77cm

    View Answer play_arrow
  • question_answer53)     The potential difference between the target and the cathode of an X-ray tube is 50 kV and the current in tube is 20 mA. Only 1% of the total energy supplied is emitted as X-radiation. What is the maximum frequency of the emitted radiation?

    A)
     \[1.2\times {{10}^{17}}Hz\]       

    B)
     \[1.2\times {{10}^{19}}Hz\]

    C)
     \[6\times {{10}^{15}}Hz\]           

    D)
     \[2.4\times {{10}^{18}}Hz\]

    View Answer play_arrow
  • question_answer54)     In the depletion region of an unbiased\[p-n\] junction diode there are

    A)
     only electrons

    B)
     only holes

    C)
     both electrons and holes

    D)
    only fixed ions

    View Answer play_arrow
  • question_answer55)     A particle of mass\[m=5\]is moving with a uniform speed\[v=3\sqrt{2}\]in the XOY plane along the line\[y=x+4\]. The magnitude of the angular momentum of the particle about the origin is

    A)
     7.5 unit               

    B)
     60 unit

    C)
     \[40\sqrt{2}\]unit          

    D)
     zero

    View Answer play_arrow
  • question_answer56) An ionic compound has a unit cell consisting of A ions at the corners of a cube and B ions on the centres of faces of the cube. The empirical formula of the compound would be

    A)
     AB                                        

    B)
     \[{{A}_{2}}B\]

    C)
     \[A{{B}_{3}}\]                                  

    D)
     \[{{A}_{3}}B\]

    View Answer play_arrow
  • question_answer57)     Acetylene hydrocarbons are acidic because

    A)
     acetylene contains least number of hydrogen atoms

    B)
     acetylene has only one hydrogen atom at each carbon atom

    C)
     acetylene belongs to the class of alkynes with formula\[{{C}_{n}}{{H}_{2n-2}}\]

    D)
     sigma electron density of\[C-H\]bond in acetylene is nearer a carbon which has 50% \[s-\]character

    View Answer play_arrow
  • question_answer58)     Which of the following does not contain \[-COOH\]group?

    A)
     Aspirin

    B)
     Benzoic acid

    C)
     Picric acid

    D)
     All have \[-COOH\]group

    View Answer play_arrow
  • question_answer59)     Only an aldehyde having ... can undergoes the aldol condensation.

    A)
     at least one alpha H atom

    B)
     at least one beta H atom

    C)
     no alpha H atom

    D)
     an aromatic ring

    View Answer play_arrow
  • question_answer60)     When sodium is added to ethanol,

    A)
     no action occurs

    B)
    \[{{H}_{2}}\]is formed

    C)
    \[NaO{{C}_{2}}{{H}_{5}}\]and\[{{O}_{2}}\]are formed

    D)
     \[NaO{{C}_{2}}{{H}_{5}}\]and\[{{H}_{2}}\]are formed

    View Answer play_arrow
  • question_answer61)     The gold number of some colloidal solutions are given below
    Colloidal solution Gold number
    A 0.01
    B 2.5
    C 20
    The protective nature of these colloidal solutions follows the order

    A)
     \[C>B>A\]         

    B)
     \[~A>B>C\]

    C)
     \[A=B=C\]         

    D)
     \[B>A>C\]

    View Answer play_arrow
  • question_answer62)     The reaction of\[HBr\]with\[C{{H}_{3}}\overset{\begin{smallmatrix}  C{{H}_{3}} \\  | \end{smallmatrix}}{\mathop{C}}\,=C{{H}_{2}}\]in the presence of peroxide will give

    A)
     \[C{{H}_{3}}\underset{\begin{smallmatrix}  | \\  C{{H}_{3}} \end{smallmatrix}}{\mathop{C}}\,BrC{{H}_{3}}\]             

    B)
    \[C{{H}_{3}}C{{H}_{2}}C{{H}_{2}}C{{H}_{2}}Br\]

    C)
     \[C{{H}_{3}}\overset{\begin{smallmatrix}  C{{H}_{3}} \\  | \end{smallmatrix}}{\mathop{C}}\,HC{{H}_{2}}Br\]          

    D)
     \[C{{H}_{3}}C{{H}_{2}}\overset{\begin{smallmatrix}  C{{H}_{3}} \\  | \end{smallmatrix}}{\mathop{C}}\,HC{{H}_{3}}\]

    View Answer play_arrow
  • question_answer63)     The alcohol manufactured from water gas is

    A)
     ethanol              

    B)
     methanol

    C)
     isobutanol         

    D)
     butanol

    View Answer play_arrow
  • question_answer64)     The iodoform test is not given by

    A)
     \[C{{H}_{3}}\underset{\begin{smallmatrix}  | \\  OH \end{smallmatrix}}{\mathop{C}}\,HC{{H}_{3}}\]    

    B)
     \[C{{H}_{3}}\overset{\begin{smallmatrix}  O \\  |\,| \end{smallmatrix}}{\mathop{C}}\,C{{H}_{2}}C{{H}_{3}}\]

    C)
     \[C{{H}_{3}}C{{H}_{2}}OH\]

    D)
     \[C{{H}_{3}}C{{H}_{2}}\overset{\begin{smallmatrix}  O \\  |\,| \end{smallmatrix}}{\mathop{C}}\,C{{H}_{2}}C{{H}_{3}}\]

    View Answer play_arrow
  • question_answer65)     When 3,3-dimethyl-2-butanol is heated with \[{{H}_{2}}S{{O}_{4}},\]the major product obtained is

    A)
     2,2-dimethyl-l-butene

    B)
     2,3-dimethyl-l-butene

    C)
     2,3-dimethyl-2-butene

    D)
    \[cis\]and trans isomers of 2,3-dimethyl-2- butane

    View Answer play_arrow
  • question_answer66)     The correct order of basicity of amines in water is

    A)
     \[{{(C{{H}_{3}})}_{2}}NH>C{{H}_{3}}N{{H}_{2}}>{{(C{{H}_{3}})}_{3}}N\]

    B)
     \[C{{H}_{3}}N{{H}_{2}}>{{(C{{H}_{3}})}_{2}}NH>{{(C{{H}_{3}})}_{3}}N\]

    C)
     \[{{(C{{H}_{3}})}_{3}}N>{{(C{{H}_{3}})}_{2}}NH>C{{H}_{3}}N{{H}_{2}}\]

    D)
     None of the above

    View Answer play_arrow
  • question_answer67)     The number of nodes present in radial wave function of 3d orbital is

    A)
     1                                            

    B)
     2

    C)
     0                                            

    D)
     3

    View Answer play_arrow
  • question_answer68)     Which of the following has largest negative electron gain enthalpy?

    A)
     \[F\]                                    

    B)
     \[Cl\]

    C)
     \[Br\]                                  

    D)
    \[I\]

    View Answer play_arrow
  • question_answer69)     Bromine belongs to period

    A)
     third                                    

    B)
     fourth

    C)
     fifth                                     

    D)
     second

    View Answer play_arrow
  • question_answer70)     \[PC{{l}_{5}}\]molecule has the following geometry

    A)
     trigonal bipyramidal

    B)
     octahedral.

    C)
     square planar

    D)
     planar triangular

    View Answer play_arrow
  • question_answer71)     In an octahedral structure, the pair of d-orbitals involved in\[{{d}^{2}}s{{p}^{3}}\]hybridisation is

    A)
     \[{{d}_{{{x}^{2}}-{{y}^{2}}}},{{d}_{xz}}\]                              

    B)
     \[{{d}_{{{z}^{2}}}},{{d}_{zx}}\]

    C)
     \[{{d}_{xy}},{{d}_{yz}}\]                              

    D)
     \[{{d}_{{{x}^{2}}-{{y}^{2}}}},{{d}_{{{z}^{2}}}}\]

    View Answer play_arrow
  • question_answer72)     For the electrode reaction,\[{{M}^{n+}}(aq)+n{{e}^{-}}\xrightarrow[{}]{{}}M(s)\]Nernst equation is

    A)
     \[E=E{}^\circ +\frac{RT}{nF}\log \frac{1}{[{{M}^{n+}}]}\]

    B)
     \[E{}^\circ =E+RT\,in\,[{{M}^{n+}}]\]

    C)
     \[E=E{}^\circ -\frac{RT}{nF}\,in\frac{1}{\,[{{M}^{n+}}]}\]

    D)
     \[\frac{E}{E{}^\circ }=\frac{RT}{nF}\,in[{{M}^{n+}}]\]

    View Answer play_arrow
  • question_answer73)     Two liquids A and B boil at\[145{}^\circ C\]and \[190{}^\circ C\] respectively. At\[80{}^\circ C\]which of them has higher vapour pressure?

    A)
     Liquid A

    B)
     Liquid B

    C)
     Both have equal vapour pressure

    D)
     None of the above

    View Answer play_arrow
  • question_answer74)     What is the effect of carbon dioxide in water on corrosion?

    A)
     Increase rusting of iron

    B)
     Decrease rusting of iron

    C)
     Does not affect

    D)
     None of the above

    View Answer play_arrow
  • question_answer75)     Delocalised electrons are present in

    A)
     1,3-butadiene                 

    B)
     \[{{C}_{6}}{{H}_{6}}\]

    C)
     1,3,5-hexatriene            

    D)
     All of these

    View Answer play_arrow
  • question_answer76)     The chemical name of isoprene is

    A)
     2-methyl-1,3-butadiene

    B)
     2-chloro-1,3-butadiene

    C)
     2-methoxypropene

    D)
     None of the above

    View Answer play_arrow
  • question_answer77)     Assertion/Reason Type Question  Answer Codes (i) Both Assertion  and Reason (R) are correct and (R) is the correct explanation of (ii) Both  and (R) are correct but (R) is not the correct explanation of (iii)  is correct but (R) is incorrect (iv)  is incorrect but (R) is correct Assertion  Methyl cyanide on reaction with\[LiAl{{H}_{4}}\]does not form ethyl amine. Reason (R) Acidic hydrolysis, of JRCN forms\[RCOOH\].

    A)
     (i)                                          

    B)
     (ii)

    C)
     (iii)                        

    D)
     (iv)

    View Answer play_arrow
  • question_answer78)     The Birch reduction of toluene gives

    A)
       

    B)
     

    C)
      

    D)
     

    View Answer play_arrow
  • question_answer79)     Given below is the sketch of a plant for carrying out a process. Name the process occurring in the above plant.

    A)
     Reverse osmosis            

    B)
     Osmosis

    C)
     Diffusion            

    D)
     None of these

    View Answer play_arrow
  • question_answer80)     In a reaction between A and B, the initial rate of reaction\[({{r}_{o}})\]was measured for different initial concentrations of A and B as given below
    \[A/mol\,{{L}^{-1}}\] 0.20 0.20 0.40
    \[B/mol\,{{L}^{-1}}\] 0.30 0.10 0.05
    \[{{r}_{0}}/\]\[mol\,{{L}^{-1}}{{s}^{-1}}\] \[5.07\times {{10}^{-5}}\] \[5.07\times {{10}^{-5}}\] \[1.43\times {{10}^{-4}}\]
    What is the order of the reaction with respect to A and B?

    A)
     2.5, 1.0                

    B)
     1.5,0

    C)
     2.5,0                                    

    D)
     1.5, 1

    View Answer play_arrow
  • question_answer81)     The most stable conformation of 1,2-diphenyl ethane is

    A)
     

    B)
     

    C)
                         

    D)
     

    View Answer play_arrow
  • question_answer82)     According to nuclear reaction,\[_{4}Be+_{2}^{4}He\xrightarrow[{}]{{}}_{6}^{12}C+_{0}^{1}n,\]the mass number of Be atom is

    A)
     4                                            

    B)
     8

    C)
     6                                            

    D)
     9

    View Answer play_arrow
  • question_answer83)     Formula of asbestos is

    A)
     \[{{[M{{g}_{3}}S{{i}_{4}}{{O}_{10}}{{(OH)}_{2}}]}_{n}}\]

    B)
     \[C{{a}_{2}}M{{g}_{5}}{{(S{{i}_{4}}{{O}_{11}})}_{2}}{{(OH)}_{2}}\]

    C)
     \[CaMg{{(Si{{O}_{3}})}_{2}}\]

    D)
     \[C{{a}_{3}}S{{i}_{3}}{{O}_{9}}\]

    View Answer play_arrow
  • question_answer84)     Which of the following resonating structures is not correct for\[C{{O}_{2}}\]?

    A)
     \[\overset{\bullet \,\,\bullet }{\mathop{{}_{\bullet }^{\bullet }O}}\,=C=\overset{\bullet \,\bullet }{\mathop{O_{\bullet }^{\bullet }}}\,\] 

    B)
     \[\underset{\bullet \,\,\,\bullet }{\mathop{^{-}\overset{\bullet \,\,\bullet }{\mathop{{}_{\bullet }^{\bullet }O}}\,}}\,=C=\overset{+}{\mathop{O_{\bullet }^{\bullet }}}\,\]

    C)
     \[\underset{\,\,\,\,\,\,\bullet \,\,\bullet }{\mathop{\overset{+}{\mathop{{}_{\bullet }^{\bullet }O}}\,}}\,-C\equiv \underset{\bullet \,\bullet }{\mathop{O{{_{\bullet }^{\bullet }}^{-}}}}\,\]            

    D)
     \[\overset{+}{\mathop{{}_{\bullet }^{\bullet }O}}\,\equiv C-\overset{\bullet \,\bullet }{\mathop{\underset{\bullet \,\bullet }{\mathop{O{{_{\bullet }^{\bullet }}^{-}}}}\,}}\,\]

    View Answer play_arrow
  • question_answer85)     At\[25{}^\circ C,\]3 g of a solute A in 100 mi/of an aqueous solution gave an osmotic pressure of 2.5 atmosphere. What is the molar mas5 of solute?

    A)
     293

    B)
     239

    C)
     392                                       

    D)
     932

    View Answer play_arrow
  • question_answer86)     The correct IUPAC name of\[C{{H}_{3}}C{{H}_{2}}CH(C{{H}_{3}})CH{{({{C}_{2}}{{H}_{5}})}_{2}}\]is

    A)
     4-ethyl-3-methylhexane

    B)
     3-ethyl-4-metbylhexane

    C)
     3-methy-4-ethyihexane

    D)
     3-iso-pentylpropane

    View Answer play_arrow
  • question_answer87)                                     Reduction of carbonyl compounds to alkanes with\[N{{H}_{2}}N{{H}_{2}}\]and\[NaOH\]is called

    A)
     Ponndrof verley reduction

    B)
     Clemmensens reduction

    C)
     Wurtz reaction                       

    D)
     Wolff-Kishner reduction

    View Answer play_arrow
  • question_answer88)     General formula of alkynes is

    A)
     \[{{C}_{n}}{{H}_{2n}}\]                

    B)
     \[{{C}_{n}}{{H}_{2n+2}}\]

    C)
     \[{{C}_{2n+2}}{{H}_{n}}\]                           

    D)
     \[{{C}_{n}}{{H}_{2n-2}}\]

    View Answer play_arrow
  • question_answer89)     Alkaline\[KMn{{O}_{4}}\]oxidises acetylene to

    A)
     acetic acid         

    B)
     ethyl alcohol

    C)
     ethylene glycol               

    D)
     oxalic acid

    View Answer play_arrow
  • question_answer90)     Dehydration of alcohol is an example of

    A)
     redox reaction

    B)
     elimination reaction

    C)
     substitution reaction

    D)
     addition reaction

    View Answer play_arrow
  • question_answer91)     Gun metal is

    A)
     \[Cu,Zn,Sn\]                     

    B)
     \[Cu,Zn,Ni\]

    C)
     \[Cu,Zn,P\]                       

    D)
     \[C,N,Fe\]

    View Answer play_arrow
  • question_answer92)     How many moles of nitrogen are needed to produce 8.2 moles of ammonia by reaction with hydrogen?

    A)
     2.1                                        

    B)
     3.1

    C)
     3.2                                        

    D)
     4.1

    View Answer play_arrow
  • question_answer93)     The temperature of a gas in a closed container is\[27{}^\circ C\]. If the temperature is raised to\[327{}^\circ C,\]the pressure exerted is

    A)
     reduced to half

    B)
     doubled

    C)
     reduced to one third

    D)
     cannot be predicted

    View Answer play_arrow
  • question_answer94)     Molar heat capacity of ethanol is\[110.4\text{ }J{{K}^{-1}}\]. Its specific heat capacity is

    A)
     2.4                                        

    B)
     55.2

    C)
     5.078                                   

    D)
     110.4

    View Answer play_arrow
  • question_answer95)     For the water gas reaction,\[C(s)+{{H}_{2}}O(g)CO(g)+{{H}_{2}}O(g)\]the standard Gibbs energy at 1000 K is\[-8.1\text{ }kJ\,mo{{l}^{-1}}\]. What is its equilibrium constant?

    A)
     2.60                                      

    B)
     4.62

    C)
     2.64                                      

    D)
     None of these

    View Answer play_arrow
  • question_answer96)     For the reaction,\[2NO(g)+C{{l}_{2}}(g)2NOCl(g)\]which is true?                 

    A)
     \[{{K}_{p}}={{K}_{C}}\times RT\]             

    B)
     \[{{K}_{p}}={{K}_{C}}{{(RT)}^{2}}\]

    C)
     \[{{K}_{p}}=\frac{{{K}_{C}}}{RT}\]                           

    D)
     \[{{K}_{p}}=\frac{{{K}_{C}}}{{{(RT)}^{2}}}\]

    View Answer play_arrow
  • question_answer97)     Oxidation number of\[Mn\]in\[MnO_{4}^{-}\]ion is

    A)
     \[+1\]                                  

    B)
     \[-7\]

    C)
     \[-1\]                                   

    D)
     \[+7\]

    View Answer play_arrow
  • question_answer98)     Which of the following is not alkali metal?

    A)
    \[Na\]                                  

    B)
    \[Fr\]

    C)
    \[Ca\]                                   

    D)
    \[K\]

    View Answer play_arrow
  • question_answer99)     Inert pair effect is predominant in

    A)
    \[~Si\]                                  

    B)
    \[Pb\]

    C)
    \[Ge\]                                  

    D)
    \[Sn\]   

    View Answer play_arrow
  • question_answer100)     How many o and n bonds are there in the molecule of tetracyanoethene? \[{{(NC)}_{2}}C=C{{(CN)}_{2}}\]

    A)
     \[9\sigma ,7\pi \]                           

    B)
     \[5\sigma ,9\pi \]

    C)
     \[5\sigma ,8\pi \]                           

    D)
     \[9\sigma ,9\pi \]

    View Answer play_arrow
  • question_answer101)     Baeyers reagent is

    A)
     aqueous \[KMn{{O}_{4}}\]

    B)
     neutral\[KMn{{O}_{4}}\]

    C)
     alkaline\[KMn{{O}_{4}}\]

    D)
     aqueous bromine water

    View Answer play_arrow
  • question_answer102)     Percentage of lead in lead pencile is

    A)
     zero                                     

    B)
     20

    C)
     80                                         

    D)
     60

    View Answer play_arrow
  • question_answer103)     Low spin complex is formed by

    A)
    \[s{{p}^{3}}{{d}^{2}}\]hybridization

    B)
    \[s{{p}^{3}}d\]hybridization

    C)
    \[{{d}^{2}}s{{p}^{3}}\]hybridization

    D)
    \[s{{p}^{3}}\]hybridization

    View Answer play_arrow
  • question_answer104)     Hybrid state of central oxygen atom in ether is

    A)
     \[sp\]                                  

    B)
     \[s{{p}^{2}}\]

    C)
     \[s{{p}^{3}}d\]                                

    D)
     \[s{{p}^{3}}\]

    View Answer play_arrow
  • question_answer105)     Clemmensen reduction of a ketone is carried out in the presence of

    A)
     \[LiAl{{H}_{4}}\]in ether

    B)
     \[Zn-Hg\]with\[HCl\]

    C)
     glycol with \[KOH\]

    D)
    \[{{H}_{2}}\]with\[Pd\]as catalyst

    View Answer play_arrow
  • question_answer106)     The electrophile involved in the nitration of benzene is

    A)
    \[NO\]                                 

    B)
     \[NO_{2}^{-}\]

    C)
    \[N{{O}_{2}}\]                                  

    D)
     \[NO_{2}^{+}\]

    View Answer play_arrow
  • question_answer107)     In the reaction,\[{{C}_{6}}{{H}_{5}}-{{N}^{+}}\equiv NC{{l}^{-}}+{{H}_{3}}P{{O}_{2}}+{{H}_{2}}O\xrightarrow[{}]{{}}?\]the product formed will be

    A)
     \[{{C}_{6}}{{H}_{5}}OH\]                             

    B)
     \[{{C}_{6}}{{H}_{6}}\]

    C)
     \[{{C}_{6}}{{H}_{5}}Cl\]

    D)
     \[{{C}_{6}}{{H}_{5}}-{{C}_{6}}{{H}_{5}}\]

    View Answer play_arrow
  • question_answer108)     Amylopectin is a polymer of

    A)
     \[\beta -D-\]glucose

    B)
     \[\alpha -D-\]glucose

    C)
     \[\beta -D-\]fructose

    D)
    \[\alpha -D-\]mannose

    View Answer play_arrow
  • question_answer109)     Across the lanthanide series, the basicity of the lanthanide hydroxides

    A)
     increases

    B)
     decreases

    C)
     first increases and then decreases

    D)
     first decreases and then increases

    View Answer play_arrow
  • question_answer110)     When sodium and chlorine ion react, energy is

    A)
     released and ionic bonds are formed

    B)
     released and covalent bonds are formed

    C)
     absorbed and ionic bonds are formed

    D)
     absorbed and covalent bonds are formed

    View Answer play_arrow
  • question_answer111) The value of\[\sum\limits_{k=1}^{6}{\left( \sin \frac{2\pi k}{7}-i\cos \frac{2\pi k}{7} \right)}\]is

    A)
                                           

    B)
     \[-i\]

    C)
     1                                            

    D)
     0

    View Answer play_arrow
  • question_answer112)     If\[{{a}_{n}}\]be the nth term of an AP and if\[{{a}_{7}}=15,\]then the value of the common difference that would make\[{{a}_{2}}{{a}_{7}}{{a}_{12}}\]greatest is

    A)
     9                                            

    B)
     9/4

    C)
     0                                            

    D)
     18

    View Answer play_arrow
  • question_answer113)     If\[{{a}_{1}},{{a}_{2}},{{a}_{3}},...,{{a}_{n}}\]are in AP, where\[{{a}_{i}}>0\]for all\[i\], then value of the expression \[\frac{1}{\sqrt{{{a}_{1}}}+\sqrt{{{a}_{2}}}}+\frac{1}{\sqrt{{{a}_{2}}}+\sqrt{{{a}_{3}}}}+.....\frac{1}{\sqrt{{{a}_{n-1}}}+\sqrt{{{a}_{n}}}}\]

    A)
     \[\frac{n-1}{\sqrt{{{a}_{n}}}-\sqrt{{{a}_{1}}}}\]                

    B)
     \[\frac{n-1}{\sqrt{{{a}_{1}}}-\sqrt{{{a}_{n}}}}\]

    C)
     \[\frac{n-1}{\sqrt{{{a}_{1}}}+\sqrt{{{a}_{n}}}}\]               

    D)
     None of these

    View Answer play_arrow
  • question_answer114)     If one root of\[{{x}^{2}}-x-k=0\]is square of the other, then k is equal to

    A)
     \[2\pm \sqrt{3}\]          

    B)
     \[3\pm \sqrt{2}\]

    C)
     \[2\pm \sqrt{5}\]                           

    D)
     \[5\pm \sqrt{2}\]

    View Answer play_arrow
  • question_answer115)     The number of ways in which we can select four numbers from 1 to 30 so as to exclude every selection of four consecutive numbers is

    A)
     27378                                  

    B)
     27405

    C)
     27399                                  

    D)
     None of these

    View Answer play_arrow
  • question_answer116)     The coefficient of\[{{x}^{n}}\]in the expansion of \[{{(1-4x)}^{-1/2}}\]is

    A)
     \[\frac{(2n)!}{{{(n!)}^{2}}}\]                      

    B)
     \[\frac{2n}{{{(n!)}^{2}}}\]

    C)
     \[\frac{(2n)!}{{{n}^{2}}}\]                           

    D)
     None of these

    View Answer play_arrow
  • question_answer117)     Let A and B be symmetric matrices of the same order, then

    A)
    \[A+B\]is symmetric matrix

    B)
    \[AB-BA\]is skew symmetric matrix

    C)
    \[AB+BA\]is symmetric matrix

    D)
     All of the above

    View Answer play_arrow
  • question_answer118)     If\[\alpha ,\beta ,\gamma \]are the roots of\[{{x}^{3}}+a{{x}^{2}}+b=0,\]then  the value of\[\left| \begin{matrix}    \alpha  & \beta  & \gamma   \\    \beta  & \gamma  & \alpha   \\    \gamma  & \alpha  & \beta   \\ \end{matrix} \right|\]is

    A)
     \[-{{a}^{3}}\]                                    

    B)
     \[{{a}^{3}}-3b\]

    C)
     \[{{a}^{3}}\]                                     

    D)
     \[{{a}^{2}}-3b\]

    View Answer play_arrow
  • question_answer119)     If the axes are shifted to the point\[(1,-2)\]without     rotation     the     equation \[2{{x}^{2}}+{{y}^{2}}-4x+4y=0\]becomes

    A)
     \[2{{X}^{2}}+{{Y}^{2}}=6\]

    B)
     \[2{{X}^{2}}+{{Y}^{2}}+6=0\]

    C)
     \[{{X}^{2}}+2{{Y}^{2}}=6\]

    D)
     \[2{{X}^{2}}+{{Y}^{2}}=0\]

    View Answer play_arrow
  • question_answer120)     The lines represented by the equation \[A{{x}^{2}}+2Bxy+H{{y}^{2}}=0\]are perpendicular, if

    A)
     \[A+B=0\]         

    B)
     \[B+H=0\]

    C)
     \[A+H=0\]         

    D)
     \[AH=-1\]

    View Answer play_arrow
  • question_answer121)     The number of common tangents to the circles \[{{x}^{2}}+{{(y-1)}^{2}}=9\]and\[{{(x-1)}^{2}}+{{y}^{2}}=25\]is

    A)
     0                                            

    B)
     1

    C)
     2                                            

    D)
     3

    View Answer play_arrow
  • question_answer122)     The length of focal chord which makes an angle \[\alpha \]with the axis. of the parabola\[{{y}^{2}}=4ax\]is

    A)
     \[2a\text{ }co{{t}^{2}}\alpha \]                

    B)
     \[4a\text{ }cose{{c}^{2}}\alpha \]

    C)
     \[4a\text{ }cot\,\alpha \]            

    D)
     None of these

    View Answer play_arrow
  • question_answer123)     The point of intersection of the tangents at two points on the ellipse\[\frac{{{x}^{2}}}{{{a}^{2}}}+\frac{{{y}^{2}}}{{{b}^{2}}}=1,\]whose eccentric angles differ by a right angle lies on the ellipse

    A)
     \[\frac{{{x}^{2}}}{{{a}^{2}}}+\frac{{{y}^{2}}}{{{b}^{2}}}=2\]                         

    B)
     \[\frac{{{x}^{2}}}{{{b}^{2}}}+\frac{{{y}^{2}}}{{{a}^{2}}}=1\]

    C)
     \[\frac{{{x}^{2}}}{{{b}^{2}}}+\frac{{{y}^{2}}}{{{a}^{2}}}=2\]                         

    D)
     \[\frac{{{x}^{2}}}{{{a}^{2}}}+\frac{{{y}^{2}}}{{{b}^{2}}}=4\]

    View Answer play_arrow
  • question_answer124)     If the normal at\[\left( ct,\frac{c}{t} \right)\]on the curve\[xy={{c}^{2}}\]meets the curve again in t, then

    A)
     \[t=-\frac{1}{{{t}^{3}}}\]                             

    B)
     \[t=-\frac{1}{t}\]

    C)
     \[t=\frac{1}{{{t}^{2}}}\]                               

    D)
     \[t{{}^{2}}=-\frac{1}{{{t}^{2}}}\]

    View Answer play_arrow
  • question_answer125)     If\[f(x)=\left\{ \begin{matrix}    \frac{|x-4|}{x-4}, & x\ne 4  \\    0, & x=4  \\ \end{matrix} \right.\]then\[\underset{x\to 4}{\mathop{\lim }}\,f(x)\]is equal to

    A)
     1                                            

    B)
    \[-1\]

    C)
     0                                            

    D)
     does not exist

    View Answer play_arrow
  • question_answer126)     The equation\[\left| \begin{matrix}    x-a & x-b & x-c  \\    x-b & x-c & x-a  \\    x-c & x-a & x-b  \\ \end{matrix} \right|=0\]where a, b, c are different is satisfied by

    A)
     \[x=0\]               

    B)
     \[x=a\]

    C)
     \[x=\frac{1}{3}(a+b+c)\]             

    D)
     \[x=a+b+c\]

    View Answer play_arrow
  • question_answer127)     The function\[f(x)=\left\{ \begin{matrix}    x & for & x<1  \\    2-x & for & 1\le x\le 2  \\    -2+3x-{{x}^{2}} & for & x>2  \\ \end{matrix} \right.\]is differentiable

    A)
     at\[x=2\]and at\[x=1\]

    B)
     at\[x=2\]but not at\[x=1\]

    C)
     at\[x=1\]but not at\[x=2\]

    D)
     neither at\[x=2\]nor at\[x=1\]

    View Answer play_arrow
  • question_answer128)     Find the points on the curve\[y={{x}^{3}}-2{{x}^{2}}-x\] at which the tangent lines are parallel to the line\[y~=3x-2\]

    A)
     \[(2,2),\left( -\frac{2}{3},-\frac{14}{27} \right)\]

    B)
     \[(2,-2),\left( -\frac{2}{3},-\frac{14}{37} \right)\]

    C)
     \[(2,-2),\left( -\frac{2}{3},-\frac{14}{27} \right)\]

    D)
     None of the above:

    View Answer play_arrow
  • question_answer129)     If\[f\]and g are two increasing functions such that fog is denned, then

    A)
     fog is an increasing function

    B)
     fog is a decreasing function

    C)
     fog is neither increasing nor decreasing

    D)
     None of the above

    View Answer play_arrow
  • question_answer130)     The value of the integral\[\int{\frac{2x\,dx}{({{x}^{2}}+1)({{x}^{2}}+2)}}\]is

    A)
     \[\log |{{x}^{2}}+1|+\log |{{x}^{2}}+2|+c\]

    B)
     \[-[\log |{{x}^{2}}+1|+\log |{{x}^{2}}+2|+c\]

    C)
     \[\log |{{x}^{2}}+1|-\log |{{x}^{2}}+2|+c\]

    D)
     \[\log |{{x}^{2}}+2|-\log |{{x}^{2}}+1|+c\]

    View Answer play_arrow
  • question_answer131)     The area of the figure bounded by the curves \[y=|x-1|\]is\[y=3|x|\]

    A)
     1                                            

    B)
     2

    C)
     3                                            

    D)
     4

    View Answer play_arrow
  • question_answer132)     If the unit vectors\[\overrightarrow{a}\]and\[\overrightarrow{b}\]are inclined at an angle\[2\theta \]such that\[|\overrightarrow{a}-\overrightarrow{b}|<1\]and\[0\le \theta k,\pi \]then \[\theta \]lies in the interval

    A)
     \[\left[ 0,\frac{\pi }{6} \right)\]                

    B)
     \[\left( \frac{5\pi }{6},\pi  \right]\]

    C)
     Both  and           

    D)
     Neither (a) nor (b)

    View Answer play_arrow
  • question_answer133)     The plane\[\frac{x}{a}+\frac{y}{b}+\frac{z}{c}=1\]meets the coordinate axes at A, B and C respectively, find the equation of the sphere OABC

    A)
    \[{{x}^{2}}+{{y}^{2}}+{{z}^{2}}+ax+by+cz=0\]

    B)
    \[{{x}^{2}}+{{y}^{2}}+{{z}^{2}}-ax-by-cz=0\]

    C)
     \[{{x}^{2}}+{{y}^{2}}+{{z}^{2}}+ax-by+cz=0\]

    D)
     None of the above

    View Answer play_arrow
  • question_answer134)     A bag contains 16 coins of which two are counterfeit with heads on both sides. The rest are fair coins. One coin is selected at random from the bag and tossed. The probability of getting a head is

    A)
     \[\frac{9}{16}\]                               

    B)
     \[\frac{11}{16}\]

    C)
     \[\frac{5}{9}\]                                  

    D)
     None of these

    View Answer play_arrow
  • question_answer135)     The value of\[\sqrt{3}\cot 20{}^\circ -4\cos 20{}^\circ \]is equal to

    A)
     1                                            

    B)
    \[-1\]

    C)
     0                                            

    D)
     None of these

    View Answer play_arrow
  • question_answer136)     If\[sin\text{ }x+cosec\text{ }x=2,\]then\[{{\sin }^{n}}x+\cos e{{c}^{n}}x\]is equal to

    A)
     2                                            

    B)
     \[{{2}^{n}}\]

    C)
     \[{{2}^{n-1}}\]                                 

    D)
     \[{{2}^{n-2}}\]

    View Answer play_arrow
  • question_answer137)     The number of solutions of the equation tan \[x+sec\text{ }x=2\text{ }cos\text{ }x,\]lying in the interval \[[0,2\pi ]\]is

    A)
     0                                            

    B)
     1

    C)
     2                                            

    D)
     3

    View Answer play_arrow
  • question_answer138)     The arithmetic mean of a set of observations is \[\overline{X}\]. If each observation is divided by a and then is increased by 10, then the mean of the new series is

    A)
     \[\frac{\overline{X}}{\alpha }\]                                

    B)
     \[\frac{\overline{X}+10}{\alpha }\]

    C)
     \[\frac{\overline{X}+10\,\alpha }{\alpha }\]                       

    D)
     \[\alpha \overline{X}+10\]

    View Answer play_arrow
  • question_answer139)     If\[x\]and y are two uncorrelated variables and if \[u=x+y,v=x-y,\]then\[r(u,v)\]is equal to

    A)
     \[\frac{\sigma _{x}^{2}+\sigma _{y}^{2}}{\sigma _{x}^{2}-\sigma _{y}^{2}}\]

    B)
     \[\frac{\sigma _{x}^{2}-\sigma _{y}^{2}}{\sigma _{x}^{2}+\sigma _{y}^{2}}\]

    C)
     \[\frac{\sigma _{x}^{2}+\sigma _{y}^{2}}{{{\sigma }_{x}}{{\sigma }_{y}}}\]                          

    D)
     None of these

    View Answer play_arrow
  • question_answer140)     If\[u\]and\[v\]be the components of the resultant velocity w of a particle such that\[u=v=w,\]then the angle between the velocities is

    A)
     \[60{}^\circ \]                                  

    B)
    \[150{}^\circ \]

    C)
     \[120{}^\circ \]                                

    D)
     \[30{}^\circ \]

    View Answer play_arrow
  • question_answer141)     \[\left\{ \frac{1+\cos \frac{\pi }{8}+i\sin \frac{\pi }{8}}{1+\cos \frac{\pi }{8}-i\sin \frac{\pi }{8}} \right\}\]is equal to

    A)
     \[1+i\]                                 

    B)
    \[1-i\]

    C)
     1                                            

    D)
     \[-1\]

    View Answer play_arrow
  • question_answer142)     The arithmetic mean of two positive numbers and\[b(a>b)\]is twice their geometric mean, then\[a:b\]is

    A)
     \[2+\sqrt{3}:2-\sqrt{3}\]             

    B)
     \[7+4\sqrt{3}:7-4\sqrt{3}\]

    C)
     \[2:7+4\sqrt{3}\]                            

    D)
     \[2\sqrt{3}\]

    View Answer play_arrow
  • question_answer143)     Given that tan A and tan B are the roots of the equation\[{{x}^{2}}-px+q=0,\]the value of \[{{\sin }^{2}}(A+B)\]is

    A)
     \[\frac{{{p}^{2}}}{{{p}^{2}}+{{(1-q)}^{2}}}\]       

    B)
     \[\frac{{{p}^{2}}}{{{p}^{2}}+{{q}^{2}}}\]

    C)
     \[\frac{{{q}^{2}}}{{{p}^{2}}+{{(1-q)}^{2}}}\]       

    D)
     \[\frac{{{p}^{2}}}{{{(p+q)}^{2}}}\]

    View Answer play_arrow
  • question_answer144)     How many four letter words can be formed using the letters of the word FAILURE, so that F is included in each word?

    A)
     400                                       

    B)
     420

    C)
     460                                       

    D)
     480

    View Answer play_arrow
  • question_answer145)     Find the coefficient of\[{{x}^{4}}\]in the expansion of

    A)
     4                                            

    B)
     8

    C)
     12                                         

    D)
     None of these

    View Answer play_arrow
  • question_answer146)     If A is a skew symmetric matrix, then the matrix \[{{B}^{T}}AB\]is

    A)
     symmetric         

    B)
     skew symmetric

    C)
     cant say              

    D)
     None of these

    View Answer play_arrow
  • question_answer147)     If the lines\[x+2ay+a=0,\text{ }x+3by+b=0\]and \[x+4cy+c=0\]are concurrent, then a, b, c are in

    A)
     AP                                        

    B)
     GP

    C)
     HP                                        

    D)
     None of these

    View Answer play_arrow
  • question_answer148)     The distance between the parallel lines represented      by      the      equation \[{{x}^{2}}+6xy+9{{y}^{2}}+4x+12y-5=0\]is

    A)
     \[\frac{6}{5}\]                                  

    B)
     \[\frac{6}{\sqrt{10}}\]

    C)
     \[\frac{6}{\sqrt{5}}\]                                    

    D)
     \[\frac{3}{2}\]

    View Answer play_arrow
  • question_answer149)     If the lengths of the tangents from the point (1,2) to the circles\[{{x}^{2}}+{{y}^{2}}+x+y-4=0\]and                  \[3{{x}^{2}}+3{{y}^{2}}-x-y-\lambda =0\]are in the ratio\[4:3,\]then the value of\[\lambda \]is

    A)
     \[\frac{21}{2}\]                               

    B)
     \[\frac{21}{4}\]

    C)
     \[\frac{21}{5}\]                               

    D)
     \[\frac{21}{11}\]

    View Answer play_arrow
  • question_answer150)     If M is the foot of the perpendicular from a point P oh a parabola to its directrix and SPM is an equilateral triangle, where\[S\]is the focus, then PM is equal to

    A)
     \[a\]                                    

    B)
     \[2a\]

    C)
     \[3a\]                                  

    D)
     \[4a\]

    View Answer play_arrow
  • question_answer151)     The equations of tangents to the ellipse \[9{{x}^{2}}+16{{y}^{2}}=144\]which pass through the point \[(2,3)\]are

    A)
     \[y=3,y=x+5\]  

    B)
     \[y=3,x=3\]

    C)
     \[x=2,\text{ }y=x+5\]   

    D)
     \[y=3,\text{ }y=-x+5\]

    View Answer play_arrow
  • question_answer152)     Let                                                                                                                           and\[e\]be the eccentricities of a hyperbola and its conjugate, then\[\frac{1}{{{e}^{2}}}+\frac{1}{e{{}^{2}}}\]is equal to

    A)
     0                                            

    B)
     1

    C)
     2                                            

    D)
     None of these

    View Answer play_arrow
  • question_answer153)     If\[f(x)\]is an even differentiable function on R, then\[f(x)\]is

    A)
     an even function

    B)
     an odd function

    C)
     cant say

    D)
     cant be determined

    View Answer play_arrow
  • question_answer154)     The inverse of the function\[f(x)=\frac{{{10}^{x}}-{{10}^{-x}}}{{{10}^{x}}+{{10}^{-x}}}\]is

    A)
     \[{{\log }_{10}}(2-x)\]                   

    B)
     \[\frac{1}{2}{{\log }_{10}}\left( \frac{1+x}{1-x} \right)\]

    C)
     \[\frac{1}{2}{{\log }_{10}}(2x-1)\]           

    D)
     \[\frac{1}{4}{{\log }_{10}}\left( \frac{2x}{2-x} \right)\]

    View Answer play_arrow
  • question_answer155)     The value of the\[\underset{x\to \infty }{\mathop{\lim }}\,{{x}^{1/x}}\]is equal to

    A)
     0                                            

    B)
     1

    C)
     \[e\]                                    

    D)
     \[{{e}^{-1}}\]

    View Answer play_arrow
  • question_answer156)     If\[f(x)={{e}^{x}}g(x),g(0)=2,g(0)=1,\]then\[f(0)\]is equal to

    A)
     1                                            

    B)
     3

    C)
     2                                            

    D)
     0

    View Answer play_arrow
  • question_answer157)     The slope of the tangent to the curve \[x={{t}^{2}}+3t-8,\text{ }y=2{{t}^{2}}-2t-5\]at the point \[(2,-1)\]is

    A)
     \[\frac{22}{7}\]                               

    B)
     \[\frac{6}{7}\]

    C)
     \[-\frac{6}{7}\]                                

    D)
     None of these

    View Answer play_arrow
  • question_answer158)     The function\[f(x)={{\cot }^{-1}}x+x\]increases in the interval

    A)
     \[(1,\infty )\]                                   

    B)
     \[(-1,\infty )\]

    C)
     \[(-\infty ,\infty )\]                        

    D)
     \[(0,\infty )\]

    View Answer play_arrow
  • question_answer159)     The  maximum  slope  of the  curve \[y=-{{x}^{3}}+3{{x}^{2}}+2x-27\]is

    A)
     5                                            

    B)
     \[-5\]

    C)
     \[\frac{1}{5}\]                                  

    D)
     None of these

    View Answer play_arrow
  • question_answer160)     The integral\[\int{{{\sin }^{3}}x}{{\cos }^{3}}x\,dx\]is equal to

    A)
     \[\frac{1}{32}\left[ -\frac{3}{2}\cos 2x+\frac{1}{6}\cos 6x \right]+c\]

    B)
     \[\frac{1}{32}\left[ -\frac{3}{2}\sin 2x+\frac{1}{6}\sin 6x \right]+c\]

    C)
     \[-\frac{1}{32}\left[ -\frac{3}{2}\cos 2x+\frac{1}{6}\sin 6x \right]+c\]

    D)
     None of the above

    View Answer play_arrow
  • question_answer161)     The value of\[\int{\frac{{{e}^{x}}\,dx}{\sqrt{5-4{{e}^{x}}-{{e}^{2x}}}}}\]is equal to

    A)
     \[{{\tan }^{-1}}\left( \frac{{{e}^{x}}+2}{3} \right)+c\]

    B)
     \[{{\sin }^{-1}}\left( \frac{{{e}^{x}}+2}{3} \right)+c\]

    C)
     \[{{\cos }^{-1}}\left( \frac{{{e}^{x}}+2}{3} \right)+c\]

    D)
     None of the above

    View Answer play_arrow
  • question_answer162)     \[\int_{0}^{1}{\frac{({{x}^{\alpha }}-1)dx}{\log x}}\]equals

    A)
     \[\frac{1}{\alpha +1}\]                 

    B)
     \[\frac{1}{\alpha -1}\]

    C)
     \[\alpha -1\]                                     

    D)
     None of these

    View Answer play_arrow
  • question_answer163)     Solution of\[{{e}^{(dy/dx)}}=x+1;y(0)=5\]is

    A)
    \[y=x\text{ }log(x+1)-x-log(x+1)+5\]

    B)
    \[y=x\text{ }log(x+1)+x+log(x+1)+5\]

    C)
     \[y=x\text{ }log(x+1)-x+log(x+1)+5\]

    D)
     None of the above

    View Answer play_arrow
  • question_answer164)     For any vector\[\overrightarrow{a},|\overrightarrow{a}\times \hat{i}{{|}^{2}}+|\overrightarrow{a}\times \hat{j}{{|}^{2}}+|\overrightarrow{a}\times \hat{k}{{|}^{2}}\]is equal to

    A)
     \[|\overrightarrow{a}{{|}^{2}}\]                                             

    B)
     \[2|\overrightarrow{a}{{|}^{2}}\]

    C)
     \[3|\overrightarrow{a}{{|}^{2}}\]                          

    D)
     None of these

    View Answer play_arrow
  • question_answer165)     The shortest distance between the lines \[\overrightarrow{r}=(4\hat{i}-\hat{j})+\lambda (\hat{i}+2\hat{j}-3\hat{k})\]and \[\overrightarrow{r}=(\hat{i}-\hat{j}+2\hat{k})+\mu (2\hat{i}+4\hat{j}-5\hat{k})\]is equal to

    A)
     \[\frac{6}{\sqrt{10}}\]                  

    B)
     \[\frac{6}{\sqrt{5}}\]

    C)
     \[\frac{6}{5}\]                                  

    D)
     \[\frac{3}{5}\]

    View Answer play_arrow
  • question_answer166)     If A and B are two independent events such that\[P(\overline{A}\cap B)=\frac{2}{15}\]and\[P(A\cap \overline{B})=\frac{1}{6},\]then P is

    A)
     \[\frac{1}{5}\]                                  

    B)
     \[\frac{5}{6}\]

    C)
     \[\frac{4}{5}\]                                  

    D)
     \[\frac{7}{6}\]

    View Answer play_arrow
  • question_answer167)     Maximum and minimum values of\[6\text{ }sin\text{ }x\text{ }cos\text{ }x+4\text{ }cos\text{ }2x\]are respectively

    A)
     5 and\[-5\]

    B)
    \[2\sqrt{13}\]and\[-2\sqrt{13}\]

    C)
     10 and\[-10\]                   

    D)
     \[\frac{5}{2}\]and\[-\frac{5}{2}\]

    View Answer play_arrow
  • question_answer168)     In a\[\Delta ABC,\]if \[\frac{\cos A}{a}=\frac{\cos B}{b}=\frac{\cos C}{c}\]and the side\[a=2,\]then area of the triangle is

    A)
     1                                            

    B)
     2

    C)
     \[\frac{\sqrt{3}}{2}\]                                    

    D)
     \[\sqrt{3}\]

    View Answer play_arrow
  • question_answer169)     The equation\[{{\sin }^{-1}}x-{{\cos }^{-1}}x={{\cos }^{-1}}\left( \frac{\sqrt{3}}{2} \right)\]has

    A)
     no solution

    B)
     unique solution

    C)
     infinite number of solutions

    D)
     None of the above

    View Answer play_arrow
  • question_answer170)     A tower subtends an angle a at a point A in the plane of its base and the angle of depression of the foot of the tower at a point b ft just above A is P. Then, height of the tower is

    A)
     \[b\text{ }tan\text{ }\alpha \text{ }cot\text{ }\beta \]                  

    B)
     \[b~cot\text{ }\alpha \text{ }tan\text{ }\beta \]

    C)
     \[b\text{ }tan\text{ }\alpha \text{ }tan\text{ }\beta \]                  

    D)
     \[b\text{ }cot\text{ }\alpha \text{ }cot\text{ }\beta \]

    View Answer play_arrow

Study Package

   


You need to login to perform this action.
You will be redirected in 3 sec spinner